Is α an element of B in Rudin's Theorem 1.11?

  • Thread starter Government$
  • Start date
  • Tags
    Theorem
In summary, the theorem proves that if S is an ordered set with the least-upper-bound property and B is a non-empty subset of S that is bounded below, then the supremum of the set of all lower bounds of B exists in S and is equal to the infimum of B. However, it is not necessarily a member of B. The theorem also shows that the intersection of two closed sets can be a singleton, in which case the supremum, maximum, infimum, and minimum will all be equal. It is important to note that this only applies when the two subsets are complements in the parent set S.
  • #1
Government$
87
1
Theorem: Suppose S is an ordered set with the least-upper-bound property, B⊂S, B is not empty, and B is bounded below. Let L be the set of all lower bounds of B. Then α=supL exists in S, and α=infB.

Rudin proves that α=supL, α is an element of L and that α=infB.
For α to be sup i.e. lub it has to be in set of upper bounds. Here i quote rudin: "Since B is bounded below, L is not empty. Since L consists of exactly those y that are elemet of S which satisfy the inequality y≤x for every x from B, we see that evry x that is in B is an upper bound of L. Thus L is bounded above. Our hypothesis about S implies therefore that L has a supremum in S; call it α. If γ<α then γ is not an upper bound of L, hence γ is not an element of B. It follows that α≤x for every x in B. Thus α is an element of L. If α<β then β is not an element of of L, since α is an upper bound of L. We have shoh that α is an element of L but β is not element of L if α<β. In other words , α is lower bound of B, but β is not is not if α<β. This means that α=infB."


So since b consists of all of upper bounds of L and α is least upper bound, can i say that α is also an elemet of B?

If that is the case can i then say that maxL=supL=infB=minB?
Because α is an element of L and at the same time α=supL then maxL=α. If it were not the cas that maxL=α then there would be some other element β in L, that is greater then α. But this would contadict the fact that supL=α. Same goes for minB=α just reveresed.

Are these conclusions correct?

P.S. Why i don't have LATEX reference in editor?
 
Last edited:
Physics news on Phys.org
  • #2
Something is very wrong. Let S = (0,1) ##\cup## (2,3), B = (2,3). Now B is bounded below, eg by 0 and 1/2, but sup(L) = 2 ##\not\in## S.
 
  • #3
verty said:
Something is very wrong. Let S = (0,1) ##\cup## (2,3), B = (2,3). Now B is bounded below, eg by 0 and 1/2, but sup(L) = 2 ##\not\in## S.

If i may usk where does least-upper-bound property come into play in your counter example?
 
Last edited:
  • #4
verty said:
Something is very wrong. Let S = (0,1) ##\cup## (2,3), B = (2,3). Now B is bounded below, eg by 0 and 1/2, but sup(L) = 2 ##\not\in## S.

Since you gave counter example you assumed that S is an order set with least-upper-bound property. (Right?) If so then this means that if E is subest of S, and E is non empty and E is bounded above , then sup E exists in S.
So let S = (0,1) ##\cup## (2,3) and Let E= (2,3). E is subset of S and it is no empty and it is bounded above and more importantly supE should exists in S. But E=(2,3) fails to have sup in S. Sup of E is 3 and it is not in S, because i can come infinetly close to 3 but i will never find element that is greater then all other elements in E. Therefore S fails to be an order set with least-upper-bound property and there fore your contuer example is false. Is my reasoning correct here?
 
  • #5
Government$ said:
Since you gave counter example you assumed that S is an order set with least-upper-bound property. (Right?) If so then this means that if E is subest of S, and E is non empty and E is bounded above , then sup E exists in S.
So let S = (0,1) ##\cup## (2,3) and Let E= (2,3). E is subset of S and it is no empty and it is bounded above and more importantly supE should exists in S. But E=(2,3) fails to have sup in S. Sup of E is 3 and it is not in S, because i can come infinetly close to 3 but i will never find element that is greater then all other elements in E. Therefore S fails to be an order set with least-upper-bound property and there fore your contuer example is false. Is my reasoning correct here?

I wasn't sure what the "least upper bound property" was but I couldn't see what would make the theorem true. This time let S = (0,1] ##\cup## (2,3], B = (2,3].

Oh, I realize now what is going on. It has be given implicitly that only points of S are lower bounds. With this premise, the theorem is true.

Back to your original question, no, my counterexample: S = (0,2], B = (1,2).
 
  • #6
You should definitely look at Real Mathematical Analysis by Charles Chapman Pugh. I find it much easier to read than Rudin. I honestly don't know why Baby Rudin is preferred by so many, I feel like Baby is sort of outdated and lacks organization of thoughts in some of his proofs.

Your conclusion, however, is false. We say that a is a maximal (or minimal) element of A if a is contained in A and a=sup(A) (or a=inf(A)). I looked at the proof in Baby Rudin, and it didn't explicitly say that alpha is contained in B nor do I see how it could be inferred. Of course there are cases in which the intersection of two closed sets is nonempty and in this case, if the intersection was a singleton, then yes sup(L)=max(L)=inf(B)=min(B). Looking ahead though, to get you ready for Dedekind cuts and cut arithmetic, I think that the point of the theorem is actually to show that a supremum or infimum of a set need not be a member of the set. And this is always the case when the two subsets are complements in the parent set S. The assumption is "Let L be the set containing all lower bounds of B". This is why sup(L) is contained in L and so sup(L)=max(L)=inf(B).
 

1. What is Theorem 1.11 in "Baby Rudin" about?

Theorem 1.11 in "Baby Rudin" is about the completeness property of real numbers. It states that every nonempty subset of real numbers that is bounded above has a least upper bound, also known as a supremum.

2. How is Theorem 1.11 used in real analysis?

Theorem 1.11 is a fundamental concept in real analysis and is used to prove many important results, such as the intermediate value theorem, the extreme value theorem, and the Bolzano-Weierstrass theorem.

3. Can you provide an example of how Theorem 1.11 is applied?

Sure, let's say we have a set A = {1/n | n ∈ N}. We know that A is bounded above by 1, but does not contain its supremum as 1 is not in the set. However, using Theorem 1.11, we can prove that the supremum of A is indeed 1/n, as this value is greater than all other elements in the set and is the smallest upper bound.

4. Is Theorem 1.11 applicable to all subsets of real numbers?

No, Theorem 1.11 only applies to nonempty subsets of real numbers that are bounded above. If a subset is not bounded above, the theorem does not hold.

5. Why is Theorem 1.11 important in mathematical analysis?

Theorem 1.11 is important because it provides a foundation for many other key theorems and concepts in mathematical analysis. It also allows us to make precise statements about the behavior of real numbers and their subsets, which is essential in many areas of mathematics and its applications.

Similar threads

  • Topology and Analysis
Replies
3
Views
2K
Replies
2
Views
325
Replies
4
Views
2K
  • Introductory Physics Homework Help
Replies
3
Views
1K
  • Calculus and Beyond Homework Help
Replies
14
Views
4K
  • Set Theory, Logic, Probability, Statistics
Replies
1
Views
94
  • Topology and Analysis
Replies
1
Views
3K
  • Beyond the Standard Models
Replies
9
Views
468
  • Calculus and Beyond Homework Help
Replies
2
Views
5K
Replies
2
Views
1K
Back
Top